7
$\begingroup$

I remember years ago coming across some seemingly non-trivial (ie. non-fixed point related) limits describing to the behavior of infinitely iterated trigonometric functions, but I can't for the life of me remember how to construct the proof.

Can someone point me in the right direction?


Specifically, I want to prove the following limits:

$$ \lim _{\left|n\right|\to \infty }\sqrt{\frac{4n}{3}}\left(\sin ^{\left\{n\right\}}\left(\frac{1}{\sqrt{n}}\right)\right) = 1 $$ $$\textbf{and}$$ $$ \lim _{\left|n\right|\to \infty }\sqrt{\frac{5n}{3}}\left(\tanh ^{\left\{n\right\}}\left(\frac{1}{\sqrt{n}}\right)\right) = 1 $$




I.e. that is to say:

$$ \sin \left(\sin \left(\sin \left(\sin \left(\sin \left(\frac{1}{\sqrt{5}}\right)\right)\right)\right)\right) \cdot \sqrt{\frac{4\cdot 5}{3}} \approx 1 $$
$$ \tanh \left(\tanh \left(\tanh \left(\tanh \left(\tanh \left(\tanh \left(\frac{1}{\sqrt{6}}\right)\right)\right)\right)\right)\right)\cdot \sqrt{\frac{5\cdot 6}{3}}\approx 1 $$
$$ \operatorname{arcsinh}\left(\operatorname{arcsinh}\left(\operatorname{arcsinh}\left(\frac{1}{\sqrt{3}}\right)\right)\right)\cdot \sqrt{\frac{4\cdot 3}{3}}\approx 1 $$

... and so on, noting the absolute value in the limits.


Note on Notation:

It seems people use a variety of different notations for expressing function iteration, but I went with this one since it felt most natural: $$ f^{\left\{0\right\}}\left(x\right)=x $$ $$ f^{\left\{1\right\}}\left(x\right)=f(x) $$ $$ ... $$ $$ f^{\left\{k\right\}}\left(x\right)=f\left(f^{\left\{k-1\right\}}\left(x\right)\right)\text{ } \forall k\in \mathbb{Z} $$


This has been bugging me for a while, but I can't seem to make any substantive progress (despite several hours of unsuccessful attempts to reconstruct the proof from old notes), so I will be forever grateful if you guys can give me some guidance!

$\endgroup$
3
  • $\begingroup$ Interesting and un-intuitive to me. Your first one off-the-cuff suggests to use $\sin x \approx x$ for very small $x$, and so the iterated sine would give $\approx \sqrt{1/n}$ and the entire expression would then yield $\sqrt{4/3}$. Would be happy to see advice from more seasoned people. $\endgroup$ Commented Sep 23, 2020 at 13:17
  • $\begingroup$ Another curious limit from my notebook around this period that might help, seems to be a simplification: $$\lim_{n\to \infty }\frac{\sin ^{\left\{n^2\right\}}\left(\frac{1}{n}\right)}{\sinh ^{\left\{n^2\right\}}\left(\frac{1}{n}\right)}=\frac{1}{\sqrt{2}}$$ $\endgroup$ Commented Sep 23, 2020 at 15:38
  • $\begingroup$ just checked the $\sin()/\sinh()$ formula via taylorexpansion. Got the same result,seems to be correct. $\endgroup$ Commented Sep 26, 2020 at 17:21

2 Answers 2

7
$\begingroup$

You can compare the iteration to $x_{n+1}=x_n+ax_n^2$ or $x_{n+1}=x_n+ax_n^3$ where you get asymptotic behavior similar to the Bernoulli DE solution method, that is, consider $y_n=x_n^{-2}$ or some other suitable power. In your use case you would have to treat $x_n$ as function of $x_0$ and then insert the special $x_0$ into the asymptotic expression. See

One other method (which might also be used as refinement of the first one) is to find a conjugation map to transform the recursion into one with known behavior, see Schröder's equation, and as explored in


For the sine example you get for $x_{n+1}=\sin(x_n)=x_n-\frac16x_n^3+...$ that with $y_n=x_n^{-2}$ $$ y_{n+1}=\frac2{1-\cos(2x_n)} =\frac2{2x_n^2-\frac2{3}x_n^4+\frac4{45}x_n^6\pm...} =y_n+\frac13+\frac1{15}y^{-1}+O(y_n^{-2}) \\ \implies y_n=y_0+\frac n3+C+O(\log(3y_0+n)) $$ so that with $x_0=\frac1{\sqrt n}\implies y_0=n$ it follows that $$ \lim_{n\to\infty}\frac{y_n}{n}=\frac43 \implies \lim_{n\to\infty}\sqrt{n}x_n=\frac{\sqrt3}2 $$

In the case of the $\tanh$ iteration, the additive constant changes from $\frac13$ to $\frac23$, everything else remains largely the same, so that $\frac{y_n}n\to\frac53$.


Generalizing to $x_0=\frac{x}{\sqrt{n}}$ one finds $\frac{y_n}{n}=\frac1{x^2}+\frac1{3}+O(\frac{\log(n)}{n})$, so that $$ \lim_{n\to\infty}\sqrt{n}\sin^{\circ n}\left(\frac{x}{\sqrt n}\right)=\frac{x}{\sqrt{1+3x^2}}. $$

$\endgroup$
3
  • $\begingroup$ Is there a simpler way to understand geometrically why the ratio of sin and hyperbolic sin iterated in this way yields 1/sqrt(2)? $\endgroup$ Commented Apr 4, 2022 at 17:38
  • $\begingroup$ With $f(x)=\frac{\sin(x)}{\sinh(x)}$ you get a different class of functions, as this has a constant term. I'm not sure where you get the value from, the fixed point in $[0,1]$ is at $0.8054$, which is not $\sqrt{0.5}$. But you get $\frac{\sin^2x}{\sinh x}=x-\frac12x^3+O(x^5)$, which would indeed converge to $\frac1{\sqrt{1+2·\frac12}}$. $\endgroup$ Commented Apr 4, 2022 at 18:57
  • $\begingroup$ Thank Lutz, also for anyone curious, there are some helpful visual diagrams in jstor.org/stable/10.4169/math.mag.87.5.338 "Iteration of Sine and Related Power Series" which also goes into proving the arctan identity which clarified the tanh^{-x} case where -x is a negative integer. $\endgroup$ Commented Apr 5, 2022 at 1:35
1
$\begingroup$

Disclaimer: This isn't really an answer, but something that I tried.

I use $\sin_n$ to denote the sine function iterated $n$ times. I formulate the problem as: Show that $$ \sin_n (\frac{1}{\sqrt n}) \to \frac{\sqrt 3}{2} \frac{1}{\sqrt n}$$ I saw this post about Taylor approximation for iterated sine: here which says that $$ \sin_n(x) = x - \frac{n}{6}x^3 - \left(\frac{n}{30} - \frac{n^2}{24} \right)x^5 + \epsilon$$ I plug in $x = 1 / \sqrt n$ and get $$ \sin_n(\frac{1}{\sqrt n}) = \frac{1}{\sqrt n} \left( \frac{5}{6} - \left( \frac{\frac{4}{n} - 5}{120} \right) \right) + \epsilon$$ So as $n \to \infty$, the term inside the big brackets goes to $$ \frac{5}{6} + \frac{5}{120} = 5 \left(\frac{1}{3!} + \frac{1}{5!} \right)$$ I just make a wild guess that if more terms of the Taylor expansion are used, you going to get a pattern $$5 \left(\frac{1}{3!} + \frac{1}{5!} + \frac{1}{7!} \dotsm\right)$$ and Wolfram says this is $5(\sinh(1)-1) = 0.8760...$. Compare to $\sqrt 3 / 2 = 0.8660...$ and it seems pretty close...

$\endgroup$
2
  • $\begingroup$ Interesting, thanks for the help! I'm gonna try the same method with tanh and see if this is the way to go! $\endgroup$ Commented Sep 23, 2020 at 15:39
  • $\begingroup$ I get for the $\tanh()$-version $0.774596669241 \approx \sqrt{3/5}$ . Here I use the method of Carleman-matrices for functions which have a taylorseries expansion and which exploit the concept of simple matrix-powers for function-iterates. It's a simple method and I can put example in an answer-box if still needed. $\endgroup$ Commented Sep 26, 2020 at 9:39

You must log in to answer this question.

Start asking to get answers

Find the answer to your question by asking.

Ask question

Explore related questions

See similar questions with these tags.